LSAT and Law School Admissions Forum

Get expert LSAT preparation and law school admissions advice from PowerScore Test Preparation.

User avatar
 Dave Killoran
PowerScore Staff
  • PowerScore Staff
  • Posts: 5853
  • Joined: Mar 25, 2011
|
#82637
Complete Question Explanation

The correct answer choice is (E).

The condition in the question stem can only occur under Template #2. In that template M’s stall and G’s stall are always in the same row (the first row), and thus answer choice (E) is correct.
 LilyRose
  • Posts: 19
  • Joined: Mar 13, 2016
|
#25752
This question stem can only lead to one template:

(F, G) - M
K - J - H

But there seem to be two answers that must be true?
C. G is assigned to a lower-numbered stall than M.
(Yes, it must be true. Whether G is in 1 and F is in 2, or F is in 1 and G is in 2, G will always be lower than M, which is 3.)
E. M's stall is in the same row as G's stall.
(Again, this must be true, whether G is in 1 or 2.)

The right answer, apparently, is E. Why is C not right? Have I overlooked a possibility somewhere?
 Emily Haney-Caron
PowerScore Staff
  • PowerScore Staff
  • Posts: 577
  • Joined: Jan 12, 2012
|
#25878
Hi Lily,

The diagram I got here with the addition of this new rule is:
__ - M/ - /M (with F and G filling the empty spots)
K - J - H

That leaves the only correct answer as E.

If you can walk us through how you got to the template you used for this question, we can try to help explain a bit more!
 LilyRose
  • Posts: 19
  • Joined: Mar 13, 2016
|
#25911
your M/ made me realize my error. I had placed M in the 3rd slot only, but it could have been in 2 or 3.

I just overlooked a template when I drew my diagram. :oops: Oops.

Thank you for your response!

Get the most out of your LSAT Prep Plus subscription.

Analyze and track your performance with our Testing and Analytics Package.